Find the volume of a rotationally symmetric 3D body












0












$begingroup$



let $S$ be the set of points $(x,y,z)$ satisfying $ x^2 + y^2 + z^2 = 1 $ and $ 0leq z leq frac{1}{sqrt{2}} $
let $D$ be the $ 3 D $ body obtained by taking the union of all segments connecting $(0,0,0)$ to the points of $S$
Find the volume of the body.




Here is my trial :



Let
$g(t) = (0,0,0) + t(x,y,z)$ such that t $ in [0,1]$
by the giving $ S $ in the question we get that :
$ D =$ {$0 leq x^2 + y^2 + z^2 leq 1$}



so we want to do triple integral on $ D$ to find the volume :
$$
int_0^{2pi}
int_{pi/4}^{pi/2}
int_0^1 r^2 sin(phi) { dr} d{phi } d{theta}
$$



the thing is when i move to spherical coordinates i get different answer than the basic coordinates .



Spherical Answer : $ frac{sqrt{2}pi}{3} $



x,y,z coordinates Answer $ frac{5pi}{6sqrt{2}} $










share|cite|improve this question











$endgroup$












  • $begingroup$
    it would help if you would write in the actual integrals you are computing and the calculations. looks like an error in arithmetic likely
    $endgroup$
    – gt6989b
    Dec 24 '18 at 16:32










  • $begingroup$
    i have updated the integral
    $endgroup$
    – Mather
    Dec 24 '18 at 16:49










  • $begingroup$
    Are you sure that is the answer you obtain through the spherical integral?
    $endgroup$
    – Shubham Johri
    Dec 24 '18 at 16:50










  • $begingroup$
    i am not sure of the answer i tried to calculate the question in different ways but i failed to calculate in the spherical coordinates , and according to the answer down there the one in the x,y,z is right but the one in the spherical is wrong
    $endgroup$
    – Mather
    Dec 24 '18 at 16:51












  • $begingroup$
    @Mather The answer obtained through spherical co-ordinates is correct
    $endgroup$
    – Shubham Johri
    Dec 24 '18 at 17:12
















0












$begingroup$



let $S$ be the set of points $(x,y,z)$ satisfying $ x^2 + y^2 + z^2 = 1 $ and $ 0leq z leq frac{1}{sqrt{2}} $
let $D$ be the $ 3 D $ body obtained by taking the union of all segments connecting $(0,0,0)$ to the points of $S$
Find the volume of the body.




Here is my trial :



Let
$g(t) = (0,0,0) + t(x,y,z)$ such that t $ in [0,1]$
by the giving $ S $ in the question we get that :
$ D =$ {$0 leq x^2 + y^2 + z^2 leq 1$}



so we want to do triple integral on $ D$ to find the volume :
$$
int_0^{2pi}
int_{pi/4}^{pi/2}
int_0^1 r^2 sin(phi) { dr} d{phi } d{theta}
$$



the thing is when i move to spherical coordinates i get different answer than the basic coordinates .



Spherical Answer : $ frac{sqrt{2}pi}{3} $



x,y,z coordinates Answer $ frac{5pi}{6sqrt{2}} $










share|cite|improve this question











$endgroup$












  • $begingroup$
    it would help if you would write in the actual integrals you are computing and the calculations. looks like an error in arithmetic likely
    $endgroup$
    – gt6989b
    Dec 24 '18 at 16:32










  • $begingroup$
    i have updated the integral
    $endgroup$
    – Mather
    Dec 24 '18 at 16:49










  • $begingroup$
    Are you sure that is the answer you obtain through the spherical integral?
    $endgroup$
    – Shubham Johri
    Dec 24 '18 at 16:50










  • $begingroup$
    i am not sure of the answer i tried to calculate the question in different ways but i failed to calculate in the spherical coordinates , and according to the answer down there the one in the x,y,z is right but the one in the spherical is wrong
    $endgroup$
    – Mather
    Dec 24 '18 at 16:51












  • $begingroup$
    @Mather The answer obtained through spherical co-ordinates is correct
    $endgroup$
    – Shubham Johri
    Dec 24 '18 at 17:12














0












0








0





$begingroup$



let $S$ be the set of points $(x,y,z)$ satisfying $ x^2 + y^2 + z^2 = 1 $ and $ 0leq z leq frac{1}{sqrt{2}} $
let $D$ be the $ 3 D $ body obtained by taking the union of all segments connecting $(0,0,0)$ to the points of $S$
Find the volume of the body.




Here is my trial :



Let
$g(t) = (0,0,0) + t(x,y,z)$ such that t $ in [0,1]$
by the giving $ S $ in the question we get that :
$ D =$ {$0 leq x^2 + y^2 + z^2 leq 1$}



so we want to do triple integral on $ D$ to find the volume :
$$
int_0^{2pi}
int_{pi/4}^{pi/2}
int_0^1 r^2 sin(phi) { dr} d{phi } d{theta}
$$



the thing is when i move to spherical coordinates i get different answer than the basic coordinates .



Spherical Answer : $ frac{sqrt{2}pi}{3} $



x,y,z coordinates Answer $ frac{5pi}{6sqrt{2}} $










share|cite|improve this question











$endgroup$





let $S$ be the set of points $(x,y,z)$ satisfying $ x^2 + y^2 + z^2 = 1 $ and $ 0leq z leq frac{1}{sqrt{2}} $
let $D$ be the $ 3 D $ body obtained by taking the union of all segments connecting $(0,0,0)$ to the points of $S$
Find the volume of the body.




Here is my trial :



Let
$g(t) = (0,0,0) + t(x,y,z)$ such that t $ in [0,1]$
by the giving $ S $ in the question we get that :
$ D =$ {$0 leq x^2 + y^2 + z^2 leq 1$}



so we want to do triple integral on $ D$ to find the volume :
$$
int_0^{2pi}
int_{pi/4}^{pi/2}
int_0^1 r^2 sin(phi) { dr} d{phi } d{theta}
$$



the thing is when i move to spherical coordinates i get different answer than the basic coordinates .



Spherical Answer : $ frac{sqrt{2}pi}{3} $



x,y,z coordinates Answer $ frac{5pi}{6sqrt{2}} $







integration multivariable-calculus jacobian






share|cite|improve this question















share|cite|improve this question













share|cite|improve this question




share|cite|improve this question








edited Dec 24 '18 at 17:23









gt6989b

36.1k22557




36.1k22557










asked Dec 24 '18 at 16:27









Mather Mather

4088




4088












  • $begingroup$
    it would help if you would write in the actual integrals you are computing and the calculations. looks like an error in arithmetic likely
    $endgroup$
    – gt6989b
    Dec 24 '18 at 16:32










  • $begingroup$
    i have updated the integral
    $endgroup$
    – Mather
    Dec 24 '18 at 16:49










  • $begingroup$
    Are you sure that is the answer you obtain through the spherical integral?
    $endgroup$
    – Shubham Johri
    Dec 24 '18 at 16:50










  • $begingroup$
    i am not sure of the answer i tried to calculate the question in different ways but i failed to calculate in the spherical coordinates , and according to the answer down there the one in the x,y,z is right but the one in the spherical is wrong
    $endgroup$
    – Mather
    Dec 24 '18 at 16:51












  • $begingroup$
    @Mather The answer obtained through spherical co-ordinates is correct
    $endgroup$
    – Shubham Johri
    Dec 24 '18 at 17:12


















  • $begingroup$
    it would help if you would write in the actual integrals you are computing and the calculations. looks like an error in arithmetic likely
    $endgroup$
    – gt6989b
    Dec 24 '18 at 16:32










  • $begingroup$
    i have updated the integral
    $endgroup$
    – Mather
    Dec 24 '18 at 16:49










  • $begingroup$
    Are you sure that is the answer you obtain through the spherical integral?
    $endgroup$
    – Shubham Johri
    Dec 24 '18 at 16:50










  • $begingroup$
    i am not sure of the answer i tried to calculate the question in different ways but i failed to calculate in the spherical coordinates , and according to the answer down there the one in the x,y,z is right but the one in the spherical is wrong
    $endgroup$
    – Mather
    Dec 24 '18 at 16:51












  • $begingroup$
    @Mather The answer obtained through spherical co-ordinates is correct
    $endgroup$
    – Shubham Johri
    Dec 24 '18 at 17:12
















$begingroup$
it would help if you would write in the actual integrals you are computing and the calculations. looks like an error in arithmetic likely
$endgroup$
– gt6989b
Dec 24 '18 at 16:32




$begingroup$
it would help if you would write in the actual integrals you are computing and the calculations. looks like an error in arithmetic likely
$endgroup$
– gt6989b
Dec 24 '18 at 16:32












$begingroup$
i have updated the integral
$endgroup$
– Mather
Dec 24 '18 at 16:49




$begingroup$
i have updated the integral
$endgroup$
– Mather
Dec 24 '18 at 16:49












$begingroup$
Are you sure that is the answer you obtain through the spherical integral?
$endgroup$
– Shubham Johri
Dec 24 '18 at 16:50




$begingroup$
Are you sure that is the answer you obtain through the spherical integral?
$endgroup$
– Shubham Johri
Dec 24 '18 at 16:50












$begingroup$
i am not sure of the answer i tried to calculate the question in different ways but i failed to calculate in the spherical coordinates , and according to the answer down there the one in the x,y,z is right but the one in the spherical is wrong
$endgroup$
– Mather
Dec 24 '18 at 16:51






$begingroup$
i am not sure of the answer i tried to calculate the question in different ways but i failed to calculate in the spherical coordinates , and according to the answer down there the one in the x,y,z is right but the one in the spherical is wrong
$endgroup$
– Mather
Dec 24 '18 at 16:51














$begingroup$
@Mather The answer obtained through spherical co-ordinates is correct
$endgroup$
– Shubham Johri
Dec 24 '18 at 17:12




$begingroup$
@Mather The answer obtained through spherical co-ordinates is correct
$endgroup$
– Shubham Johri
Dec 24 '18 at 17:12










2 Answers
2






active

oldest

votes


















2












$begingroup$

The integral in cartesian co-ordinates looks like this: $I=I_1-I_2$



$displaystyle I_1=int_{-1}^1int_{-sqrt{1-y^2}}^{sqrt{1-y^2}}sqrt{1-x^2-y^2} dx dy=2pi/3\I_2=displaystyleint_{-frac1{sqrt2}}^{frac1{sqrt2}}int_{-sqrt{frac12-y^2}}^{sqrt{frac12-y^2}}sqrt{1-x^2-y^2}-sqrt{x^2+y^2} dx dy$



$I_1$ is just the volume of the hemsiphere. $I_2$ gives the volume of the inverted cone with spherical base that needs to be subtracted from $I_1$.



The answer obtained is the same as the one produced by the spherical integral: $sqrt2pi/3$.



FigureFigureFigure






share|cite|improve this answer











$endgroup$













  • $begingroup$
    i sliced the domain into circules as a function of z like the one up there
    $endgroup$
    – Mather
    Dec 24 '18 at 17:22












  • $begingroup$
    I'm adding more details, just wait a sec
    $endgroup$
    – Shubham Johri
    Dec 24 '18 at 17:23










  • $begingroup$
    The domain is a collection of annular discs parallel to the $z=0$ plane. The answer above has assumed it to be a collection of complete discs
    $endgroup$
    – Shubham Johri
    Dec 24 '18 at 17:32










  • $begingroup$
    but i dont understand why i cant slice the body into discs with radius $ 1-z^2 $
    $endgroup$
    – Mather
    Dec 24 '18 at 17:44










  • $begingroup$
    Do you know a good 3D surface plotter?
    $endgroup$
    – Shubham Johri
    Dec 24 '18 at 17:46



















2












$begingroup$

enter image description here



$$intlimits_{z=0}^{1/sqrt{2}} pi left( (1 - z^2) - z^2 right) dz = intlimits_{z=0}^{1/sqrt{2}} pi (1 - 2 z^2) dz = frac{sqrt{2} pi }{3}$$



Annular disk of inner radius $z$ and outer radius $sqrt{1 - z^2}$ and thickness $dz$ (in blue).






share|cite|improve this answer











$endgroup$













  • $begingroup$
    yes i got this answer on basic coordinates but on spherical i got diffrent one
    $endgroup$
    – Mather
    Dec 24 '18 at 16:34










  • $begingroup$
    Show your full spherical integral.
    $endgroup$
    – David G. Stork
    Dec 24 '18 at 16:34










  • $begingroup$
    i updated it without the limits
    $endgroup$
    – Mather
    Dec 24 '18 at 16:36










  • $begingroup$
    You don't have the Jacobian.
    $endgroup$
    – David G. Stork
    Dec 24 '18 at 16:36










  • $begingroup$
    the jacobian is the integrand itself $ r^2sin(phi) $ isn't it ?
    $endgroup$
    – Mather
    Dec 24 '18 at 16:38












Your Answer








StackExchange.ready(function() {
var channelOptions = {
tags: "".split(" "),
id: "69"
};
initTagRenderer("".split(" "), "".split(" "), channelOptions);

StackExchange.using("externalEditor", function() {
// Have to fire editor after snippets, if snippets enabled
if (StackExchange.settings.snippets.snippetsEnabled) {
StackExchange.using("snippets", function() {
createEditor();
});
}
else {
createEditor();
}
});

function createEditor() {
StackExchange.prepareEditor({
heartbeatType: 'answer',
autoActivateHeartbeat: false,
convertImagesToLinks: true,
noModals: true,
showLowRepImageUploadWarning: true,
reputationToPostImages: 10,
bindNavPrevention: true,
postfix: "",
imageUploader: {
brandingHtml: "Powered by u003ca class="icon-imgur-white" href="https://imgur.com/"u003eu003c/au003e",
contentPolicyHtml: "User contributions licensed under u003ca href="https://creativecommons.org/licenses/by-sa/3.0/"u003ecc by-sa 3.0 with attribution requiredu003c/au003e u003ca href="https://stackoverflow.com/legal/content-policy"u003e(content policy)u003c/au003e",
allowUrls: true
},
noCode: true, onDemand: true,
discardSelector: ".discard-answer"
,immediatelyShowMarkdownHelp:true
});


}
});














draft saved

draft discarded


















StackExchange.ready(
function () {
StackExchange.openid.initPostLogin('.new-post-login', 'https%3a%2f%2fmath.stackexchange.com%2fquestions%2f3051419%2ffind-the-volume-of-a-rotationally-symmetric-3d-body%23new-answer', 'question_page');
}
);

Post as a guest















Required, but never shown

























2 Answers
2






active

oldest

votes








2 Answers
2






active

oldest

votes









active

oldest

votes






active

oldest

votes









2












$begingroup$

The integral in cartesian co-ordinates looks like this: $I=I_1-I_2$



$displaystyle I_1=int_{-1}^1int_{-sqrt{1-y^2}}^{sqrt{1-y^2}}sqrt{1-x^2-y^2} dx dy=2pi/3\I_2=displaystyleint_{-frac1{sqrt2}}^{frac1{sqrt2}}int_{-sqrt{frac12-y^2}}^{sqrt{frac12-y^2}}sqrt{1-x^2-y^2}-sqrt{x^2+y^2} dx dy$



$I_1$ is just the volume of the hemsiphere. $I_2$ gives the volume of the inverted cone with spherical base that needs to be subtracted from $I_1$.



The answer obtained is the same as the one produced by the spherical integral: $sqrt2pi/3$.



FigureFigureFigure






share|cite|improve this answer











$endgroup$













  • $begingroup$
    i sliced the domain into circules as a function of z like the one up there
    $endgroup$
    – Mather
    Dec 24 '18 at 17:22












  • $begingroup$
    I'm adding more details, just wait a sec
    $endgroup$
    – Shubham Johri
    Dec 24 '18 at 17:23










  • $begingroup$
    The domain is a collection of annular discs parallel to the $z=0$ plane. The answer above has assumed it to be a collection of complete discs
    $endgroup$
    – Shubham Johri
    Dec 24 '18 at 17:32










  • $begingroup$
    but i dont understand why i cant slice the body into discs with radius $ 1-z^2 $
    $endgroup$
    – Mather
    Dec 24 '18 at 17:44










  • $begingroup$
    Do you know a good 3D surface plotter?
    $endgroup$
    – Shubham Johri
    Dec 24 '18 at 17:46
















2












$begingroup$

The integral in cartesian co-ordinates looks like this: $I=I_1-I_2$



$displaystyle I_1=int_{-1}^1int_{-sqrt{1-y^2}}^{sqrt{1-y^2}}sqrt{1-x^2-y^2} dx dy=2pi/3\I_2=displaystyleint_{-frac1{sqrt2}}^{frac1{sqrt2}}int_{-sqrt{frac12-y^2}}^{sqrt{frac12-y^2}}sqrt{1-x^2-y^2}-sqrt{x^2+y^2} dx dy$



$I_1$ is just the volume of the hemsiphere. $I_2$ gives the volume of the inverted cone with spherical base that needs to be subtracted from $I_1$.



The answer obtained is the same as the one produced by the spherical integral: $sqrt2pi/3$.



FigureFigureFigure






share|cite|improve this answer











$endgroup$













  • $begingroup$
    i sliced the domain into circules as a function of z like the one up there
    $endgroup$
    – Mather
    Dec 24 '18 at 17:22












  • $begingroup$
    I'm adding more details, just wait a sec
    $endgroup$
    – Shubham Johri
    Dec 24 '18 at 17:23










  • $begingroup$
    The domain is a collection of annular discs parallel to the $z=0$ plane. The answer above has assumed it to be a collection of complete discs
    $endgroup$
    – Shubham Johri
    Dec 24 '18 at 17:32










  • $begingroup$
    but i dont understand why i cant slice the body into discs with radius $ 1-z^2 $
    $endgroup$
    – Mather
    Dec 24 '18 at 17:44










  • $begingroup$
    Do you know a good 3D surface plotter?
    $endgroup$
    – Shubham Johri
    Dec 24 '18 at 17:46














2












2








2





$begingroup$

The integral in cartesian co-ordinates looks like this: $I=I_1-I_2$



$displaystyle I_1=int_{-1}^1int_{-sqrt{1-y^2}}^{sqrt{1-y^2}}sqrt{1-x^2-y^2} dx dy=2pi/3\I_2=displaystyleint_{-frac1{sqrt2}}^{frac1{sqrt2}}int_{-sqrt{frac12-y^2}}^{sqrt{frac12-y^2}}sqrt{1-x^2-y^2}-sqrt{x^2+y^2} dx dy$



$I_1$ is just the volume of the hemsiphere. $I_2$ gives the volume of the inverted cone with spherical base that needs to be subtracted from $I_1$.



The answer obtained is the same as the one produced by the spherical integral: $sqrt2pi/3$.



FigureFigureFigure






share|cite|improve this answer











$endgroup$



The integral in cartesian co-ordinates looks like this: $I=I_1-I_2$



$displaystyle I_1=int_{-1}^1int_{-sqrt{1-y^2}}^{sqrt{1-y^2}}sqrt{1-x^2-y^2} dx dy=2pi/3\I_2=displaystyleint_{-frac1{sqrt2}}^{frac1{sqrt2}}int_{-sqrt{frac12-y^2}}^{sqrt{frac12-y^2}}sqrt{1-x^2-y^2}-sqrt{x^2+y^2} dx dy$



$I_1$ is just the volume of the hemsiphere. $I_2$ gives the volume of the inverted cone with spherical base that needs to be subtracted from $I_1$.



The answer obtained is the same as the one produced by the spherical integral: $sqrt2pi/3$.



FigureFigureFigure







share|cite|improve this answer














share|cite|improve this answer



share|cite|improve this answer








edited Dec 24 '18 at 18:41

























answered Dec 24 '18 at 17:21









Shubham JohriShubham Johri

5,683918




5,683918












  • $begingroup$
    i sliced the domain into circules as a function of z like the one up there
    $endgroup$
    – Mather
    Dec 24 '18 at 17:22












  • $begingroup$
    I'm adding more details, just wait a sec
    $endgroup$
    – Shubham Johri
    Dec 24 '18 at 17:23










  • $begingroup$
    The domain is a collection of annular discs parallel to the $z=0$ plane. The answer above has assumed it to be a collection of complete discs
    $endgroup$
    – Shubham Johri
    Dec 24 '18 at 17:32










  • $begingroup$
    but i dont understand why i cant slice the body into discs with radius $ 1-z^2 $
    $endgroup$
    – Mather
    Dec 24 '18 at 17:44










  • $begingroup$
    Do you know a good 3D surface plotter?
    $endgroup$
    – Shubham Johri
    Dec 24 '18 at 17:46


















  • $begingroup$
    i sliced the domain into circules as a function of z like the one up there
    $endgroup$
    – Mather
    Dec 24 '18 at 17:22












  • $begingroup$
    I'm adding more details, just wait a sec
    $endgroup$
    – Shubham Johri
    Dec 24 '18 at 17:23










  • $begingroup$
    The domain is a collection of annular discs parallel to the $z=0$ plane. The answer above has assumed it to be a collection of complete discs
    $endgroup$
    – Shubham Johri
    Dec 24 '18 at 17:32










  • $begingroup$
    but i dont understand why i cant slice the body into discs with radius $ 1-z^2 $
    $endgroup$
    – Mather
    Dec 24 '18 at 17:44










  • $begingroup$
    Do you know a good 3D surface plotter?
    $endgroup$
    – Shubham Johri
    Dec 24 '18 at 17:46
















$begingroup$
i sliced the domain into circules as a function of z like the one up there
$endgroup$
– Mather
Dec 24 '18 at 17:22






$begingroup$
i sliced the domain into circules as a function of z like the one up there
$endgroup$
– Mather
Dec 24 '18 at 17:22














$begingroup$
I'm adding more details, just wait a sec
$endgroup$
– Shubham Johri
Dec 24 '18 at 17:23




$begingroup$
I'm adding more details, just wait a sec
$endgroup$
– Shubham Johri
Dec 24 '18 at 17:23












$begingroup$
The domain is a collection of annular discs parallel to the $z=0$ plane. The answer above has assumed it to be a collection of complete discs
$endgroup$
– Shubham Johri
Dec 24 '18 at 17:32




$begingroup$
The domain is a collection of annular discs parallel to the $z=0$ plane. The answer above has assumed it to be a collection of complete discs
$endgroup$
– Shubham Johri
Dec 24 '18 at 17:32












$begingroup$
but i dont understand why i cant slice the body into discs with radius $ 1-z^2 $
$endgroup$
– Mather
Dec 24 '18 at 17:44




$begingroup$
but i dont understand why i cant slice the body into discs with radius $ 1-z^2 $
$endgroup$
– Mather
Dec 24 '18 at 17:44












$begingroup$
Do you know a good 3D surface plotter?
$endgroup$
– Shubham Johri
Dec 24 '18 at 17:46




$begingroup$
Do you know a good 3D surface plotter?
$endgroup$
– Shubham Johri
Dec 24 '18 at 17:46











2












$begingroup$

enter image description here



$$intlimits_{z=0}^{1/sqrt{2}} pi left( (1 - z^2) - z^2 right) dz = intlimits_{z=0}^{1/sqrt{2}} pi (1 - 2 z^2) dz = frac{sqrt{2} pi }{3}$$



Annular disk of inner radius $z$ and outer radius $sqrt{1 - z^2}$ and thickness $dz$ (in blue).






share|cite|improve this answer











$endgroup$













  • $begingroup$
    yes i got this answer on basic coordinates but on spherical i got diffrent one
    $endgroup$
    – Mather
    Dec 24 '18 at 16:34










  • $begingroup$
    Show your full spherical integral.
    $endgroup$
    – David G. Stork
    Dec 24 '18 at 16:34










  • $begingroup$
    i updated it without the limits
    $endgroup$
    – Mather
    Dec 24 '18 at 16:36










  • $begingroup$
    You don't have the Jacobian.
    $endgroup$
    – David G. Stork
    Dec 24 '18 at 16:36










  • $begingroup$
    the jacobian is the integrand itself $ r^2sin(phi) $ isn't it ?
    $endgroup$
    – Mather
    Dec 24 '18 at 16:38
















2












$begingroup$

enter image description here



$$intlimits_{z=0}^{1/sqrt{2}} pi left( (1 - z^2) - z^2 right) dz = intlimits_{z=0}^{1/sqrt{2}} pi (1 - 2 z^2) dz = frac{sqrt{2} pi }{3}$$



Annular disk of inner radius $z$ and outer radius $sqrt{1 - z^2}$ and thickness $dz$ (in blue).






share|cite|improve this answer











$endgroup$













  • $begingroup$
    yes i got this answer on basic coordinates but on spherical i got diffrent one
    $endgroup$
    – Mather
    Dec 24 '18 at 16:34










  • $begingroup$
    Show your full spherical integral.
    $endgroup$
    – David G. Stork
    Dec 24 '18 at 16:34










  • $begingroup$
    i updated it without the limits
    $endgroup$
    – Mather
    Dec 24 '18 at 16:36










  • $begingroup$
    You don't have the Jacobian.
    $endgroup$
    – David G. Stork
    Dec 24 '18 at 16:36










  • $begingroup$
    the jacobian is the integrand itself $ r^2sin(phi) $ isn't it ?
    $endgroup$
    – Mather
    Dec 24 '18 at 16:38














2












2








2





$begingroup$

enter image description here



$$intlimits_{z=0}^{1/sqrt{2}} pi left( (1 - z^2) - z^2 right) dz = intlimits_{z=0}^{1/sqrt{2}} pi (1 - 2 z^2) dz = frac{sqrt{2} pi }{3}$$



Annular disk of inner radius $z$ and outer radius $sqrt{1 - z^2}$ and thickness $dz$ (in blue).






share|cite|improve this answer











$endgroup$



enter image description here



$$intlimits_{z=0}^{1/sqrt{2}} pi left( (1 - z^2) - z^2 right) dz = intlimits_{z=0}^{1/sqrt{2}} pi (1 - 2 z^2) dz = frac{sqrt{2} pi }{3}$$



Annular disk of inner radius $z$ and outer radius $sqrt{1 - z^2}$ and thickness $dz$ (in blue).







share|cite|improve this answer














share|cite|improve this answer



share|cite|improve this answer








edited Dec 24 '18 at 19:10

























answered Dec 24 '18 at 16:33









David G. StorkDavid G. Stork

12.2k41836




12.2k41836












  • $begingroup$
    yes i got this answer on basic coordinates but on spherical i got diffrent one
    $endgroup$
    – Mather
    Dec 24 '18 at 16:34










  • $begingroup$
    Show your full spherical integral.
    $endgroup$
    – David G. Stork
    Dec 24 '18 at 16:34










  • $begingroup$
    i updated it without the limits
    $endgroup$
    – Mather
    Dec 24 '18 at 16:36










  • $begingroup$
    You don't have the Jacobian.
    $endgroup$
    – David G. Stork
    Dec 24 '18 at 16:36










  • $begingroup$
    the jacobian is the integrand itself $ r^2sin(phi) $ isn't it ?
    $endgroup$
    – Mather
    Dec 24 '18 at 16:38


















  • $begingroup$
    yes i got this answer on basic coordinates but on spherical i got diffrent one
    $endgroup$
    – Mather
    Dec 24 '18 at 16:34










  • $begingroup$
    Show your full spherical integral.
    $endgroup$
    – David G. Stork
    Dec 24 '18 at 16:34










  • $begingroup$
    i updated it without the limits
    $endgroup$
    – Mather
    Dec 24 '18 at 16:36










  • $begingroup$
    You don't have the Jacobian.
    $endgroup$
    – David G. Stork
    Dec 24 '18 at 16:36










  • $begingroup$
    the jacobian is the integrand itself $ r^2sin(phi) $ isn't it ?
    $endgroup$
    – Mather
    Dec 24 '18 at 16:38
















$begingroup$
yes i got this answer on basic coordinates but on spherical i got diffrent one
$endgroup$
– Mather
Dec 24 '18 at 16:34




$begingroup$
yes i got this answer on basic coordinates but on spherical i got diffrent one
$endgroup$
– Mather
Dec 24 '18 at 16:34












$begingroup$
Show your full spherical integral.
$endgroup$
– David G. Stork
Dec 24 '18 at 16:34




$begingroup$
Show your full spherical integral.
$endgroup$
– David G. Stork
Dec 24 '18 at 16:34












$begingroup$
i updated it without the limits
$endgroup$
– Mather
Dec 24 '18 at 16:36




$begingroup$
i updated it without the limits
$endgroup$
– Mather
Dec 24 '18 at 16:36












$begingroup$
You don't have the Jacobian.
$endgroup$
– David G. Stork
Dec 24 '18 at 16:36




$begingroup$
You don't have the Jacobian.
$endgroup$
– David G. Stork
Dec 24 '18 at 16:36












$begingroup$
the jacobian is the integrand itself $ r^2sin(phi) $ isn't it ?
$endgroup$
– Mather
Dec 24 '18 at 16:38




$begingroup$
the jacobian is the integrand itself $ r^2sin(phi) $ isn't it ?
$endgroup$
– Mather
Dec 24 '18 at 16:38


















draft saved

draft discarded




















































Thanks for contributing an answer to Mathematics Stack Exchange!


  • Please be sure to answer the question. Provide details and share your research!

But avoid



  • Asking for help, clarification, or responding to other answers.

  • Making statements based on opinion; back them up with references or personal experience.


Use MathJax to format equations. MathJax reference.


To learn more, see our tips on writing great answers.




draft saved


draft discarded














StackExchange.ready(
function () {
StackExchange.openid.initPostLogin('.new-post-login', 'https%3a%2f%2fmath.stackexchange.com%2fquestions%2f3051419%2ffind-the-volume-of-a-rotationally-symmetric-3d-body%23new-answer', 'question_page');
}
);

Post as a guest















Required, but never shown





















































Required, but never shown














Required, but never shown












Required, but never shown







Required, but never shown

































Required, but never shown














Required, but never shown












Required, but never shown







Required, but never shown







Popular posts from this blog

Plaza Victoria

In PowerPoint, is there a keyboard shortcut for bulleted / numbered list?

How to put 3 figures in Latex with 2 figures side by side and 1 below these side by side images but in...